practice naplex 2015

160
Which of the following antibiotics do not require a dosage adjustment when administer Answer A Oxacillin and amoxicillin B Nafcillin and dicloxacillin !icarcillin and oxacillin " Nafcillin and ticarcillin # $iperacillin and dicloxacillin %uestion& "uring the hospitali'ation( )*+s pain is well-controlled on morphine , mg . %/ hours dose b2 3450 Which Norco dosing regimen should the pharmacist recommend? Answer A Norco 46,34 mg ever2 73 hours B Norco 46,34 mg ever2 / hours Norco 8046,34 mg ever2 9 hours " Norco 7:6,34 mg ever2 9 hours # Norco 7:6,34 mg ever2 / hours f 84: m; of a 345 <v6v= solution of meth2l salic2late is diluted to a ,::: m; soluti sign0 A patient is using Opana #> tablets0 Which of the following statements concerning Opa Answer A Opana #> should not be ta1en with food0 B f Opana #> is ingested with alcohol there will be reduced absorption of the drug( Opana #> can be used in patients with moderate-to-severe liver impairment0 " Opana #> is an antagonist at the mu opioid receptor0 # !he chemical name is ox2codone0 Alexandra is a /9 2ear old female being seen at the methadone clinic0 @he is not enjo drug overdose0 @he is full of sadness( grief( and anxiet2 about the future0 Alexand Aleve0 Which of her medications could be contributing to her depressive s2mptoms? Answer A lonidine and Aleve B >emeron and bisacod2l lonidine and $eg- ntron " $eg- ntron and >emeron # "ocusate and bisacod2l A new drug for stro1e prevention( ;O!B @!( was studied0 !he trial enrolled 3(:93 pa aspirin arm( p-value C :0:340 What was the absolute ris1 reduction of stro1e in this 'ero when the answer is less than one0

Upload: starobin

Post on 03-Nov-2015

144 views

Category:

Documents


3 download

TRANSCRIPT

Sheet1Which of the following antibiotics do not require a dosage adjustment when administered to a patient with moderate-to-severe renal impairment?AnswerA Oxacillin and amoxicillinB Nafcillin and dicloxacillinC Ticarcillin and oxacillinD Nafcillin and ticarcillinE Piperacillin and dicloxacillinBQuestion: During the hospitalization, MG's pain is well-controlled on morphine 3 mg IV Q4 hours. The team has started discharge planning and asked the pharmacist to convert the morphine to Norco and reduce the dose by 25%. Which Norco dosing regimen should the pharmacist recommend?

AnswerA Norco 5/325 mg every 12 hoursB Norco 5/325 mg every 4 hoursC Norco 7.5/325 mg every 6 hoursD Norco 10/325 mg every 6 hoursE Norco 10/325 mg every 4 hoursdIf 750 mL of a 25% (v/v) solution of methyl salicylate is diluted to a 3000 mL solution, what will be the percentage strength of the 3 liter solution (v/v)? Enter the number only. Do not enter the percentage sign.6.25A patient is using Opana ER tablets. Which of the following statements concerning Opana ER is correct?AnswerA Opana ER should not be taken with food.B If Opana ER is ingested with alcohol there will be reduced absorption of the drug, causing lack of analgesic efficacy.C Opana ER can be used in patients with moderate-to-severe liver impairment.D Opana ER is an antagonist at the mu opioid receptor.E The chemical name is oxycodone.aAlexandra is a 46 year old female being seen at the methadone clinic. She is not enjoying her usual daily activities and is having trouble getting out of bed since her boyfriend recently passed away from a drug overdose. She is full of sadness, grief, and anxiety about the future. Alexandra takes the following medications daily: Remeron, bisacodyl, docusate, clonidine, lorazepam, methadone, Peg-Intron and Aleve. Which of her medications could be contributing to her depressive symptoms? AnswerA Clonidine and AleveB Remeron and bisacodylC Clonidine and Peg-IntronD Peg-Intron and RemeronE Docusate and bisacodylcA new drug for stroke prevention, CLOTBUST, was studied. The trial enrolled 2,062 patients. In the CLOTBUST arm, 29 out of 1,042 patients had a stroke compared with 54 patients out of 1,020 patients in the aspirin arm, p-value = 0.025. What was the absolute risk reduction of stroke in this trial? Round to nearest tenth. Enter the number only. Do not enter units, percentage sign, or commas. Include a leading zero when the answer is less than one.2.5 Select the correct application site and the maximum duration of a single application of the Transderm Scop patch:AnswerA Upper torso; 2 daysB Abdomen; 2 daysC Behind the ear; 3 daysD Hip line; 3 daysE Buttock; 2 dayscA patient with a deep vein thrombosis has been using warfarin for four months. The INR is stable around 2.5. Which of the following statements is correct?AnswerA If cimetidine is added, the INR would increase.B If trimethoprim/sulfamethoxazole is added, the INR would decrease.C If carbamazepine is added, the INR would increase.D If phenobarbital is added, the INR would increase.E If phenytoin is added, the INR would increase.aJames is a 67 year old male who frequents the pharmacy. His past medical history includes a myocardial infarction 3 years ago. Current medications include: isosorbide dinitrate, Plavix, Toprol and Altace. Today he brings in a prescription for Nexium. What action should the pharmacist take?AnswerA Fill the prescription as written.B Contact the prescriber about the interaction with Plavix. This combination could cause increased toxicity from the Plavix.C Contact the prescriber about the interaction with Plavix. This combination could cause the Plavix to have reduced efficacy.D Contact the prescriber about the interaction with Altace. This combination could cause an unsafe increase in serum potassium.E Fill the prescription as written. Advise the patient to watch for signs of bleeding.cQuestionMesalamine is available in which of the following formulations? (Select ALL that apply.)AnswerA Dipentum tabletsB Asacol-HD tabletsC Asacol suppositoriesD Rowasa enemaE Rowasa suppositoriesBDA study reports that of 52 patients who had a colonoscopy, the complication rate was 7.7% (95% Confidence Interval = 5.2% - 17.5%). What does this reported confidence interval indicate?AnswerA There is 95% confidence that the "true" complication rate is between 5.2% and 17.5%.B The "true" complication rate is between 5.2% and 17.5%, 95% of the time.C There is a 5% chance that the patient will have no complications.D There is a 95% chance that the patient will have a complication.E There is a 5% chance that the patient will have a complication.aAn 83 year old female has just lost her husband. With the grief she is enduring and stress from family coming for the funeral, she is overcome with worry. Her physician would like to prescribe her a benzodiazepine for short-term use. Which agent would be the safest recommendation for this patient?AnswerA Serax (oxazepame)B HalcionC ValiumD KlonopinE LibriumAA female patient receiving chemotherapy to treat multiple myeloma has had her labs drawn. The complete white blood cell count (WBC) with differential and basal metabolic panel were ordered. The oncologist has indicated that the patient should not get her next round of chemotherapy if the ANC is < 500 cells/mm. Calculate the patients ANC based on the laboratory values provided below. Enter the number only. Do not enter units or commas. Include a leading zero when the answer is less than one.

Labs:Na (mEq/L) = 129 (135 145) WBC (cells/mm3) = 6.7 (4 11 x 10^3) K (mEq/L) = 3.5 (3.5 5) Hgb (g/dL) = 13.4 (13.5 18 male, 12 16 female)Cl (mEq/L) = 103 (95 103) Hct (%) = 40.1 (38 50 male, 36 46 female)HCO3 (mEq/L) = 24 (24 30) Plt (cells/mm3) = 202 (150 450 x 10^3)BUN (mg/dL) = 12 (7 20) PMNs (%) = 5 (45 73)SCr (mg/dL) = 0.9 (0.6 1.3) Bands (%) = 2 (3 5) Glucose (mg/dL) = 118 (100 125) Eosinophils (%) = 3 (0 5) Basophils (%) = 0 (0 1) Lymphocytes (%) = 29 (20 40) Monocytes (%) = 2 (2 8)469A 72 year old female had a stroke. The medical resident wants to give alteplase. Which of the following statements is false?AnswerA A CT scan must confirm the absence of a hemorrhagic stroke before alteplase is given.B Treatment should be initiated within 3 hours of symptom onset.C Treatment should be initiated within 3 hours of patient arriving at the emergency department.D Intracranial hemorrhage is a serious side effect of alteplase therapy.E The maximum dose is 90 mg administered by intravenous infusion.cA 72 year old female had a stroke. The medical resident wants to give alteplase. Which of the following statements is false?AnswerA A CT scan must confirm the absence of a hemorrhagic stroke before alteplase is given.B Treatment should be initiated within 3 hours of symptom onset.C Treatment should be initiated within 3 hours of patient arriving at the emergency department.D Intracranial hemorrhage is a serious side effect of alteplase therapy.E The maximum dose is 90 mg administered by intravenous infusion.cPaulette has received a prescription for Forteo. Which statement concerning Forteo is correct?AnswerA This medicine should be used for a maximum of 3-5 years.B The generic name of Forteo is denosumab.C The same drug is available for treatment of hypercalcemia of malignancy as the branded product Xgeva.D Inject the medication using the subcutaneous injection pen each day, while sitting in a location where you can rest comfortably, such as on a bed or sitting in a comfortable chair.E The pen is good for 60 days; any medication remaining in the pen after this time must be discarded.dWhich of the following statements about pharmacogenomics are correct? (Select ALL that apply.)AnswerA Pharmacogenomics is personalized medicine, where drugs are designed for individual patients based on their phenotype.B Pharmacogenomic testing can help predict what type of response a patient is likely to have to a drug.C Codeine is converted to morphine by CYP 2D6, therefore poor metabolizers will have an increased risk of morphine toxicity.D Patients with alleles 2C92 and 2C93 are more likely to bleed on warfarin therapy.E Patients who are poor metabolizers of CYP 2C19 are not good candidates to receive Plavix.bde Elaine has NYHA functional class III systolic heart failure. She needs to use an additional drug to lower her blood pressure. She is currently taking Aceon and Lasix. Which of the following medications should be added for her blood pressure?AnswerA DiltiazemB MonoprilC CarvedilolD AmlodipineE ValsartancWhat type of information can be found in The Red Book?AnswerA Health information for international travelersB Therapeutic equivalenceC Average wholesale priceD Drug interactionsE Pediatric dosesc Counseling points on the use of Foradil should include the following: (Select ALL that apply.)AnswerA Use for maintenance only. Do not use for acute attacks.B Long-acting bronchodilators, such as Foradil, are associated with an increased risk of asthma-related deaths.C This medication should be used once daily upon first awakening.D Capsules may be kept at room temperature for up to 4 months.E Leave the capsule in the sealed pack until just before use.abdeWhich of the following statements concerning hypertensive urgency is accurate?AnswerA Acute organ damage must be present for this classification.B An appropriate treatment for hypertensive urgency is hydrochlorothiazide.C An appropriate treatment for hypertensive urgency is oral clonidine.D An appropriate treatment for hypertensive urgency is SL nifedipine.E Intravenous administration of an antihypertensive is required.cMaya is a four-year old girl with a fever of 101.7 degrees Fahrenheit. She weighs 26.4 pounds. Maya has a history of chronic otitis media infections, mild asthma with sensitivity to dust and pet hair and two recent incidents of conjunctivitis. The physician has recommended acetaminophen to be dosed at 10 mg/kg Q8H. The pharmacy has a formulation of acetaminophen suspension that contains 160 mg/5 mL. Select the correct dose, in milligrams, to be given every 8 hours from the formulations in stock. Do not round the answer. Enter the number only. Do not enter units or commas.120A patient in the critical care unit is receiving Precedex for sedation. Which of the following statements concerning Precedex is correct?AnswerA It is reconstituted in dextrose solutions only.B Precedex should only be given if a patient is also receiving a paralytic.C Precedex is an alpha2-adrenergic antagonist.D The maximum infusion duration is 48 hours.E Patients are more easily arousable and alert when stimulated, compared to propofol.eQuestionThe PDE-5 inhibitors are contraindicated with which of the following drugs? (Select ALL that apply.)AnswerA MinitranB ImdurC NifedipineD IsordilE NitrofurantoinabdWhich of the following statins should not be taken with grapefruit juice? (Select ALL that apply.)AnswerA MevacorB PravacholC LescolD CrestorE ZocoraeWhich of the following counseling statements concerning Concerta is correct?AnswerA The drug is available in a granule formulation if the child has difficulty swallowing the capsule.B If your child has difficulty swallowing, the tablet may be crushed and mixed with applesauce.C Your child's appetite may increase, therefore, give the dose in the morning after breakfast.D The drug is available in an OROS formulation that provides an immediate release portion with longer-acting portions.E Give the dose in the evening so that most of the side effects are experienced while the child is asleep.d Lisa, a 30 year-old female, is getting her first job as a hospital pharmacist since completing school. She is excited about the opportunity, but is not excited about getting shots. Lisa has no known medical conditions except for occasional acne and menstrual cramps. She will be working as a specialty pharmacist in the infectious diseases unit. Lisa states that she completed her hepatitis vaccine series (3-doses during pharmacy school, with evidence of immunity). Her mother reports that Lisa received the childhood vaccines recommended by the pediatrician. She had chickenpox as a child. What other vaccines should Lisa receive now? (Select ALL that apply.)AnswerA The influenza vaccine, seasonally, inactivated shot onlyB The influenza vaccine, seasonally, live or inactivatedC Tdap vaccine, if no record of previous vaccination, and Td every 10 years thereafter.D MenactraE Hib vaccinebcdQuestionChief Complaint: It hurts when I breathe in

History of Present Illness: SP is a 79 year old Caucasian male who presents to the emergency room on 2/11/15 because he is having increasing pain on inspiration over the past month. He is accompanied by his son and daughter who are very concerned because he "smokes cigars constantly". His past medical history is significant for COPD, hypertension, and myocardial infarction with stenting 5 years ago. He drinks 1-2 alcoholic drinks per night and smokes several cigars per day.

Allergies: penicillin

Medications: Toprol XL 50 mg daily, hydrochlorothiazide 12.5 mg daily, Zestril 10 mg daily, Spiriva 18 mcg inhaled daily, and Crestor 20 mg daily.

Physical Exam / Vitals: Height: 59 Weight: 185 pounds BP: 162/81 mmHg HR: 96 BPM RR: 14 BPM Temp: 100.6F Pain: 4/10 O2 Sat: 91% on room air

Gen: Pleasant elderly gentleman, in obvious pain from breathing. Holding side of chest.HEENT: no allergic symptoms, no nasal discharge. Coughing up sputum occasionally during exam.Heart: Regular rate and rhythm, no murmurLungs: Some wheezing bilaterally, some crackles on left.Ext: No skin lesions, good peripheral pulses bilaterally

Labs on 2/11/15:Na (mEq/L) = 136 (135 145) WBC (cells/mm3) = 13.3 (4 11 x 10^3) K (mEq/L) = 3.7 (3.5 5) Hgb (g/dL) = 16.3 (13.5 18 male, 12 16 female)Cl (mEq/L) = 101 (95 103) Hct (%) = 44.9 (38 50 male, 36 46 female)HCO3 (mEq/L) = 25 (24 30) Plt (cells/mm3) = 426 (150 450 x 10^3)BUN (mg/dL) = 22 (7 20) AST (IU/L) = 54 (10 40) SCr (mg/dL) = 1.6 (0.6 1.3) ALT (IU/L) = 43 (10 40) Glucose (mg/dL) = 115 (100 125) Albumin (g/dL) = 3.4 (3.5 5) Ca (mg/dL) = 8.3 (8.5 10.5) BNP (ng/L) = 137 (< 100 ng/L) Mg (mEq/L) = 1.8 (1.3 2.1) PO4 (mg/dL) = 4.5 (2.3 4.7) Tests on 2/11/15: Chest X-ray: 4 small nodules noted in left lower lobe. No infiltrate. Recommend aspiration or biopsy to determine source. Likely cancer.EKG: Normal sinus rhythm, no ST or T wave changes.

Plan:Admit to floor for management and further work-up of lung nodules.

Question: What is the correct interpretation of SP's elevated BNP level?

AnswerA Increased BNP is specific for heart failure. This patient has heart failure.B Increased BNP is specific for lung pathology. It correlates with the patient's lung nodules and COPD.C BNP will be permanently increased in patients who have had a myocardial infarction or stent. It can not be interpreted in this patient.D Increased BNP indicates cardiac stress, but it is not specific to heart failure or heart disease.E BNP is the most reliable marker for diagnosing a STEMI. This patient is having a STEMI.dA drug is categorized by the FDA as Pregnancy Category C. Select the correct meaning of this classification.AnswerA Studies in animals or humans show fetal abnormalities; use in pregnancy is contraindicated.B Positive evidence of fetal risk is available, but the benefits may outweigh the risk in life-threatening or serious disease.C Animal reproduction studies have failed to demonstrate a risk to the fetus and there are no adequate and well-controlled studies in pregnant women.D Animal reproduction studies have shown an adverse effect on the fetus and there are no adequate and well-controlled studies in humans, but potential benefits may warrant use of the drug in pregnant women despite potential risks.E Controlled studies in animals and women have shown no risk in the 1st trimester, and possible fetal harm is remote.dQuestionThe specific gravity of a saline solution is 1.152 at room temperature. A prescription calls for 28 grams of the saline solution. How many milliliters of the solution are needed for the prescription? Round to the nearest tenth. Enter the number only. Do not enter units.

24.3QuestionA 45 year old patient is receiving the PN formula below. How many total calories are provided by this PN? Round to the nearest whole number. Enter the number only. Do not enter units or commas.

Item/Quantities:

Dextrose 312 gramsAmino acids 62 gramsSodium chloride 32 mEqSodium acetate 12 mEqPotassium 20 mEqMagnesium sulfate 10 mEqPhosphate 21 mmolCalcium 2.2 mEqMultivitamins (MVI) 10 mLTrace elements 1 mLFamotidine 10 mgRegular insulin 10 units

Sterile water qs ad 875 mLAnswer1309Which of the following patients, if they have not received the vaccine previously, meet criteria to receive the pneumococcal vaccine (Pneumovax)? (Select ALL that apply.)AnswerA A 10 month old baby with renal insufficiency due to a congenital defect.B A 26 month old child with asthma.C A 79 year old male with COPD.D A 50 year old female smoker.E An otherwise healthy 75 year old male who has never received this vaccine.bcdeA patient is diagnosed with a Non-ST Segment Elevation Myocardial Infarction (NSTEMI). Which of the following are components of the standard NSTEMI treatment?AnswerA Nitrates, meperidine, GP IIb/IIIa receptor antagonistsB Oxygen, nitrates, GP IIb/IIIa receptor agonistsC Anticoagulants, P2Y12 inhibitors, fibrinolyticsD Oxygen, lorazepam, morphineE Anticoagulants, P2Y12 inhibitors, morphineeThe pharmacist advises the nurse on safe intravenous administration of phenytoin. Which of the following are correct statements? (Select ALL that apply.)AnswerA Phenytoin is rarely used in the emergency department due to the narrow spectrum of seizure coverage.B Intravenous phenytoin can cause arrhythmias if it is infused too quickly.C Elderly patients are more susceptible to the side effects of intravenous phenytoin compared to younger patients.D Intravenous phenytoin can cause dangerous elevations in blood pressure if infused too rapidly.E The maximum infusion rate is 50 milligrams per minute.bce Angelique is a 62 year-old African American female with hypertension. Her last BP reading was 163/99 mmHg and she is willing to be started on medication. According to JNC 8, which of the following represent this patient's blood pressure goal?AnswerA < 120/80 mmHgB < 130/80 mmHgC < 130/90 mmHgD < 140/80 mmHgE < 150/90 mmHgeBen is a 77 year old male with Parkinson disease. Ben is using carbidopa/levodopa 25/250 TID. He has been experiencing "wearing off" and the physician has prescribed ropinirole 1 mg TID. Ben's wife brings the new prescription into the pharmacy. What recommendation should the pharmacist discuss with Ben's physician?AnswerA The dose of ropinirole is too low.B The dose of ropinirole is too high.C Ropinirole is a replacement for Sinemet; the Sinemet should be discontinued.D Ropinirole is reserved primarily for younger patients. Elderly patients should not receive it, because they cannot easily tolerate the anticholinergic side effects.E It would be preferable to prescribe pramipexole.bA concerned mother has arrived at the pharmacy with a prescription for Vyvanse for her nine-year-old son. Which statement is correct concerning Vyvanse?AnswerA Vyvanse is a pro-drug of dexmethylphenidate.B Vyvanse capsules may be opened to dissolve the powder in yogurt, water or orange juice.C Vyvanse is available as an extended-release capsule and as a transdermal patch.D Vyvanse causes less tics than the active metabolite, methylphenidate.E Vyvanse is DEA Category III and is preferable by some physicians.bA oral medication is dosed 675 mg BID. After a 675 mg oral dose, the resulting AUC is 42 mg/L x hr. Bioavailability of the oral formulation is 50%. What is the clearance of this medication? Round to the nearest whole number. Enter the number only. Do not enter units or commas.8Sylvie has been prescribed Celebrex for osteoarthritis. She takes ramipril 10 mg daily for hypertension and atorvastatin 20 mg daily for high cholesterol. She is allergic to sulfamethoxazole (wheezing, bronchoconstriction) and penicillin (rash). Which of the following statements is correct?AnswerA Celebrex contains meloxicam, which has the highest COX-2 selectivity.B Sylvie should not take atorvastatin due to the stated sulfa allergy.C The recommended dose of Celebrex for osteoarthritis is 20 mg BID or 40 mg daily.D Celebrex has a decreased risk of cardiovascular complications compared to other NSAIDs and for this reason it is often used in elderly patients with co-morbid conditions.E Celebrex is indicated for various conditions, including rheumatoid arthritis pain and acute pain.eA pharmacist received a prescription for azithromycin suspension 200 mg/5 mL, Sig: 10 mg/kg on Day 1, then 5 mg/kg days 2-5, for a child weighing 44 pounds. The pharmacist did not have the 200 mg/5 mL strength on hand and the antibiotic needed to be started immediately. The pharmacist gave the mother a 15 mL bottle of azithromycin 100 mg/5 mL strength to use until more azithromycin suspension could be obtained. The mother calls the pharmacy to say she has just finished the temporary supply. The bottle is now empty. Assuming the the medication was measured and administered correctly, how many more days of treatment remain? Enter the number only. Do not enter units or commas.3 Craig has treatment-resistant depression. He has failed reasonable trials of doxepin, escitalopram, fluoxetine and venlafaxine. Which of the following statements apply to treatment-resistant depression? (Select ALL that apply.)AnswerA In some patients, a dosage increase is all that is required if the dose remains reasonable for the particular agent.B Lithium is occasionally used for antidepressant augmentation.C Abilify is an antipsychotic used as an add-on to the antidepressant for treatment-resistant depression.D It is never appropriate to combine different antidepressants.E Nefazodone is sometimes used as an augmentation agent, but has a risk of renal toxicity.abcWhich of the following statements concerning meningococcal vaccination is correct?AnswerA Saudi Arabia requires proof of vaccination with Menveo or Menomune prior to travel at certain times of the year.B Saudi Arabia requires vaccination with Twinrix prior to travel at certain times of the year.C The meningitis belt of Australia requires vaccination with Menveo or Menomune prior to travel at certain times of the year.D The meningitis belt of Australia requires vaccination with Pneumovax prior to entry.E The meningitis belt of Africa requires vaccination with Pneumovax 23 year-round.aWhich of the following drugs are glycoprotein IIb/IIIa receptor antagonists?AnswerA Abciximab, ticagrelor, clopidogrelB Prasugrel, eptifibatide, abciximabC Eptifibatide, tirofiban, abciximabD Tirofibran, eptifibatide, ticagrelorE Clopidogrel, alteplase, tirofibancJasmine is a thin, white female who has been taking prednisone 10 mg daily for several months. The pharmacist understands the many risks associated with long-term prednisone use and reviews the patient profile to check for which of the following recommended agent/s?AnswerA DramamineB Calcium and Vitamin DC Ortho EvraD MirapexE Vitamin B6bQuestionTom has end stage renal disease (ESRD) and uses many medications, including PhosLo. Which of the following statements about PhosLo is true?AnswerA PhosLo is a second line therapy for hyperphosphatemiaB PhosLo should be taken 30 mins before mealsC PhosLo can cause an increase in serum calcium levelsD PhosLo is a synthetic polymerE PhosLo can cause an increase in serum phosphate levelscA pharmacist receives a prescription for Vigamox. The pharmacist should scan the profile for the following condition in which this drug would be indicated?AnswerA Elevated intraocular pressureB Interstitial nephritisC Urinary tract infectionD Bacterial conjunctivitisE CellulitisdLabs on 2/10/15:Na (mEq/L) = 137 (135 145) WBC (cells/mm3) = 14.2 (4 11 x 10^3) K (mEq/L) = 3.5 (3.5 5) Hgb (g/dL) = 13.6 (13.5 18 male, 12 16 female)Cl (mEq/L) = 99 (95 103) Hct (%) = 41.1 (38 50 male, 36 46 female)HCO3 (mEq/L) = 28 (24 30) Plt (cells/mm3) = 301 (150 450 x 10^3)BUN (mg/dL) = 18 (7 20) AST (IU/L) = 29 (10 40) SCr (mg/dL) = 1.2 (0.6 1.3) ALT (IU/L) = 25 (10 40) Glucose (mg/dL) = 254 (100 125) Albumin (g/dL) = 3.1 (3.5 5) Ca (mg/dL) = 7.8 (8.5 10.5) A1C (%) = 10.2 Mg (mEq/L) = 1.1 (1.3 2.1) PO4 (mg/dL) = 2.3 (2.3 4.7) Tests on 2/10/15: Radiology of left foot: soft tissue swelling, but no evidence of bone involvement. Negative for osteomyelitis.

Plan:Admit to floor for incision and drainage of left foot wound, management of infection, and management of chronic conditions.

Question: What is MG's corrected calcium level?

AnswerA 7.1 mg/dLB 8.52 mg/dLC 9.24 mg/dLD 10.28 mg/dLE 10.9 mg/dLbA pharmacist is preparing a piggyback infusion of the antibiotic Zosyn for a patient in the unit. Select the correct statement concerning Zosyn.AnswerA This medication contains ticarcillin and the beta-lactamase inhibitor clavulanate.B This is a beta-lactam antibiotic with beta lactamase inhibitor that covers Pseudomonas aeruginosa.C This drug contains the cephalosporin ceftriaxone.D This medication is one of the cephalosporins that does not require a reduced dosage for renal impairment.E This is a cephalosporin that covers MRSA.b A female patient with advanced breast cancer will be receiving a highly emetogenic chemotherapy regimen. She will be given an antiemetic regimen 30 minutes prior to the infusion. Which of the following combinations would be reasonable?AnswerA Aprepitant + Prednisone + LorazepamB Aprepitant + Methylprednisolone + MetoclopramideC Aprepitant + Methylprednisolone + CompazineD Aprepitant + Prednisone + CompazineE Aprepitant + Dexamethasone + OndansetroneQuestion: SP has a biopsy of the lung nodules and is diagnosed with small cell lung cancer (SCLC). The cancer center has had good success in limited stage SCLC with radiation and chemotherapy with cisplatin and etoposide. Where in the cell cycle do cisplatin and etoposide work?

AnswerA Cisplatin, M-phase; Etoposide, S-phaseB Cisplatin, Non-cell cycle specific; Etoposide, M-phaseC Cisplatin, S-phase; Etoposide, G2 and S-phaseD Cisplatin, Non-cell cycle specific; Etoposide, G2 and S-phaseE Cisplatin, G2 and S-phase; Etoposide, Non-cell cycle specificdCarmen has had difficult sleeping since she changed the hours of her work shift. She asked her physician for a sleeping pill and was given a prescription for Ambien. Which of the following statements concerning Ambien is correct?AnswerA This class of medications act as agonists at the serotonin receptor.B Ambien can cause somnolence and ataxia.C Ambien is available over-the-counter in a lower dosage formulation.D Ambien, compared to other hypnotics, has a lower risk of parasomnias.E The dosing for the sublingual product Intermezzo is 3.5 mg QHS for females.bChief Complaint: I'm out of refills on my inhaler and having trouble with my breathing

History of Present Illness: DD is a 24 year old Caucasian female who presents to the emergency room on 1/29/15 because she ran out of refills on her inhaler about a week ago. Her past medical history is significant for type 1 diabetes and asthma. She lives with her son. She does not drink alcohol and does not smoke tobacco. She occasionally smokes marijuana.

Allergies: penicillin

Medications: Advair Diskus 250/50 mcg 1 inhalation BID, Mircette 1 tab daily, NovoLIN 70/30 14 units BID, and an OTC multivitamin daily.

Physical Exam / Vitals: Height: 52 Weight: 118 pounds BP: 118/74 mmHg HR: 95 BPM RR: 19 BPM Temp: 98.6F Pain: 0/10 O2 Sat: 92% on room air

Gen: Thin female, very anxiousHEENT: no nasal discharge, mild erythema of throatHeart: Regular rate and rhythmLungs: Wheezing heard bilaterallyExt: No skin lesions, good peripheral pulses bilaterally

Labs on 1/29/15:Na (mEq/L) = 142 (135 145) WBC (cells/mm3) = 11.7 (4 11 x 10^3) K (mEq/L) = 4.2 (3.5 5) Hgb (g/dL) = 14.3 (13.5 18 male, 12 16 female)Cl (mEq/L) = 102 (95 103) Hct (%) = 43.7 (38 50 male, 36 46 female)HCO3 (mEq/L) = 25 (24 30) Plt (cells/mm3) = 402 (150 450 x 10^3)BUN (mg/dL) = 7 (7 20) AST (IU/L) = 18 (10 40) SCr (mg/dL) = 0.7 (0.6 1.3) ALT (IU/L) = 22 (10 40) Glucose (mg/dL) = 176 (100 125) Albumin (g/dL) = 4.7 (3.5 5) Ca (mg/dL) = 9.2 (8.5 10.5) A1C (%) = 8.7 Mg (mEq/L) = 2.2 (1.3 2.1) PO4 (mg/dL) = 8.6 (2.3 4.7) Tests on 1/29/15: Chest X-ray: no infiltrate

Plan:Admit to floor for asthma exacerbation.

Question: The pharmacist and diabetic educator meet with DD. Her insurance covers long-acting and rapid-acting insulin pens and DD is excited to gain better control of her diabetes. What dose of Levemir is equivalent to her current daily dose of NPH? Round to the nearest unit of insulin.

AnswerA 8 unitsB 11 unitsC 14 unitsD 20 unitsE 28 units

dRudy, a 52 year-old male, approaches the pharmacy counter asking for bandages to cover a wound on his foot. He stepped on a nail while tearing down his old garden shed. He does not remember which childhood vaccines he received. Which vaccine should the pharmacist recommend?AnswerA DTaPB DTC TdD TdapE None of the aboveaHelen has been taking warfarin 5 mg daily for the past 6 months for atrial fibrillation. All of her INR values have been within the normal therapeutic range (INR 2-3). The reading today is 1.7. What should the pharmacist recommend?AnswerA Continue taking warfarin 5 mg daily. Re-check the INR within 1-2 weeks.B Increase the warfarin dose by 25%.C Hold warfarin until the INR is therapeutic. Re-start at 25% of the previous dose.D Administer Vitamin K 5 mg orally. Re-start warfarin at 25% of the previous dose.E Send the patient to the emergency department to assess for likely bleeding.aQuestionA pharmacist has received a prescription for Gleevec. What indication should the pharmacist look for on the patient's profile? AnswerA Pulmonary fibrosisB CancerC OsteoarthritisD HyponatremiaE HypernatremiabAn 80-year-old male has just had a major surgery and is at high risk for bleeding.

Allergies: NKDA

Past medical history: Hyperlipidemia, hypertension, mitral valve replacement

The patient was started on enoxaparin before the surgery and the physician wishes to restart it post-surgery due to the history of the mitral valve replacement. When is it safe to restart the enoxaparin?AnswerA Inform the prescriber that it would be preferable to use clopidogrel for this indication.B Inform the prescriber that it would be preferable to use dofetilide for this indication.C The enoxaparin should be restarted immediately after the surgery if patient is hemodynamically stable.D The enoxaparin should be restarted 24 hours after the surgery if the patient is hemodynamically stable.E The enoxaparin should be restarted 48 - 72 hours after the surgery if the patient is hemodynamically stable.eWhich of the following chemotherapy drugs has the highest emetogenic risk?AnswerA BleomycinB EtoposideC ImatinibD VincristineE CisplatineThe pharmacist needs to add an additional 300 calories to a preparation of parenteral nutrition. If the pharmacist is using a 10% lipid emulsion, how many milliliters are required to meet the remaining calories? Round to the nearest milliliter. Enter the number only. Do not enter units or commas.273Susie is a six year old girl with asthma. The physician calls the pharmacy to ask for the correct dose of montelukast for this patient. Which of the following recommendations is correct?AnswerA A 4 mg dose of oral granules in the morning.B A 4 mg dose of oral granules in the evening.C A 5 mg chewable tablet once daily in the evening.D A 5 mg chewable tablet taken BID.E A 10 mg tablet once daily in the evening.c Esther has just entered a skilled nursing facility. She has a diagnosis of Parkinson disease, hypertension, occasional constipation and difficulty sleeping. Which of the following medications in her profile may worsen her movement disorder?AnswerA Hydrochlorothiazide 25 mg QAMB Metoclopramide 10 mg QIDC Caribopa/Levidopa 25/100 TIDD Lorazepam 0.5 mg QHS PRN insomniaE Docusate sodium 1-2 capsules QHS PRN constipationbAn 89 kg female patient is being transferred from the cardiac catheterization suite to the cardiac ICU with orders for an Integrilin drip to run at 2 mcg/kg/min for 18 hours. The bolus was already administered. The hospital has 100 mL vials of Integrilin 0.75 mg/mL. If the pharmacist sends a 100 mL vial, how many hours will the vial last? Round to the nearest hour. Enter only the number. Do not enter units. 7Paul is a 62 year-old male who will begin treatment with AndroGel 1.62%. Which of the following counseling statements is correct?AnswerA Apply to the upper arms and shoulders but not to the abdomen.B Apply to the upper arms, shoulders or abdomen each morning before getting dressed.C If a woman or child comes in contact with the gel wash hands quickly with alcohol gel.D The pump does not need to be primed.E The use of AndroGel can improve symptoms of prostate enlargement but it is not indicated for this purpose.aAn elderly female is asking about the risk of developing venous thromboembolism (VTE). Which of the following is not a risk factor for VTE?AnswerA Selective estrogen receptor modulating drugsB Angina, chronic or variantC CancerD ImmobilityE Major traumabWhich of the following agents contains a long-acting beta agonist? (Select ALL that apply.)AnswerA Spiriva HandiHalerB Breo ElliptaC Arcapta NeohalerD SymbicortE Tudorza Pressairbcd The mother of a 3 year-old girl brings you a prescription for an antibiotic suspension to treat acute otitis media. What is the drug of choice for this condition?AnswerA Cephalexin 50 mg/kg/day, taken once dailyB Cephalexin 50 mg/kg/day, divided BIDC Azithromycin 30 mg/kg/day, taken once dailyD Amoxicillin 90 mg/kg/day, divided BIDE Amoxicillin 90 mg/kg, divided BIDdA 44 year old male with an obstructive bowel disorder and impaired renal clearance has been NPO for 10 days. Parenteral nutrition is ordered that contains 1200 mL of D70W, 500 mL of Aminosyn 18%, and 10 mL of multivitamin for infusion daily. Prior to calculating the electrolytes to be added, the pharmacist will calculate the nitrogen intake contained in each bag. Calculate the daily grams of nitrogen the patient will be receiving from the parenteral nutrition. Round to the nearest whole number. Enter the number only. Do not enter units or commas. 14 Lorcaserin is indicated for weight loss. Which of the following statements concerning lorcaserin is correct? AnswerA The brand name is Myrbetriq.B Lorcaserin is Pregnancy Category D.C Lorcaserin is a serotonin (5-HT2c) receptor agonist.D The manufacturer is required to conduct Phase IV monitoring for cognitive effects in patients using lorcaserin.E Lorcaserin is a stimulant agent.cWhich of the following is a strong inducer of the CYP 3A4 enzyme? AnswerA ErythromycinB NizoralC NorvirD CalanE DilantineOlivia is a young girl weighing 44 pounds. She is to receive amoxicillin 90 mg/kg/day to be administered Q 8 hours for treatment of acute otitis media. The pharmacy has amoxicillin 250 mg/5 mL suspension in stock. The pharmacist will instruct the parents on how many mL of the suspension to draw up in an oral syringe. How many mL per dose should be administered? Enter the number only. Do not enter units.12 Nelson is a 57 year-old African American male with hypertension and type 2 diabetes. What is an appropriate starting class of blood pressure lowering agents according to the JNC 8 guideline?AnswerA Angiotensin Receptor BlockerB Calcium Channel BlockerC ACE InhibitorD Beta-BlockerE NitratebChoose the best definition of a cohort study.AnswerA A study that follows a group of patients with some commonality over a given time period.B A study used to estimate the relationship between an outcome of interest and population variables as they exist at a particular time.C A study in which the participants receive each treatment in a sequential order.D A pivotal clinical trial comparing a new drug to a placebo treatment group.E A pivotal clinical trial comparing a new drug to a the standard treatment used for that condition.aWhich of the following infectious disease matches (disease to vector) is correct?AnswerA Malaria, transmitted by the Anopheles mosquitoB Dengue, transmitted by the Tsetse flyC African sleeping sickness, transmitted by fecal-contaminated food or waterD Yellow fever, transmitted by infected dog or cat biteE Japanese encephalitis, transmitted by fecal-contaminated food or wateraA patient gave the pharmacist a prescription for Aranesp. The pharmacist should look for which of the following conditions on the patient's chart?AnswerA Prolactin elevationB Emphysema secondary to COPDC End stage renal diseaseD Human immunodeficiency virusE Guilain-Barre syndromecQuestionMaria is a 65 year old post-menopausal sedentary female with a family history of breast cancer. She was recently diagnosed with osteoporosis and wants to know if she can use raloxifene. Which statement concerning raloxifene is correct?AnswerA Raloxifene increases the risk of breast cancer; it is best avoided in patients with family history of this type of cancer.B Raloxifene can increase the bad cholesterol; the cholesterol panel should be checked prior to use.C Raloxifene is often used in women at risk of breast cancer, but it can cause dangerous blood clots.D The brand name is Prezista.E Raloxifene, similar to estrogen, has the additional benefit of reducing hot flashes.cWhich of the following antibiotic suspensions requires refrigeration?AnswerA CephalexinB AzithromycinC ClindamycinD Sulfamethoxazole/TrimethoprimE ClarithromycinaA clinical trial is published about a new drug, NOPAINZ, for pain reduction. There are 200 patients randomized; 100 patients to NOPAINZ and 100 patients to placebo. The study found that pain was alleviated in 85 patients taking NOPAINZ and in 45 patients taking placebo. Calculate the relative risk reduction (RRR) in this trial. Round to 2 decimal places. Enter the relative risk reduction as a decimal and not a percentage. Enter the number only. Do not enter units or commas. Include a leading zero when the answer is less than one. 0.73A patient is prescribed Atripla. What are the components of Atripla?AnswerA Efavirenz, etravirine, delavirdineB Efavirenz, elvitegravir, rilpivirineC Emtricitabine, rilpivirine, tenofovirD Emtricitabine, efavirenz, tenofovirE Emtricitabine, tenofovirdA patient has the following lab values. Calculate the patient's anion gap. Enter the number only. Do not enter units.

Labs:Na (mEq/L) = 131 (135 145) K (mEq/L) = 3.1 (3.5 5) Cl (mEq/L) = 99 (95 103) HCO3 (mEq/L) = 20 (24 30) BUN (mg/dL) = 12 (7 20) SCr (mg/dL) = 0.9 (0.6 1.3) Glucose (mg/dL) = 121 (100 125) 12Drug Z was studied for the prevention of migraines in a pilot study. There were 243 patients in the study and 126 received Drug Z while 117 received placebo. Of these, 15 patients on Drug Z and 32 patients on placebo developed a migraine. How many patients need to be treated to prevent 1 migraine? Enter the number only. 7Which of the following drugs can cause hypoglycemia?AnswerA FluoroquinolonesB Beta agonistsC Protease inhibitorsD DiazoxideE Thiazide diureticsaQuestion: In an attempt to optimize MG's medical management, his medical team calculates his estimated 10-year ASCVD risk to be 36%. His lipid panel is as follows: TC 247, LDL 167, HDL 36, and TG 160. Which of the following recommendations is consistent with the 2013 ACC/AHA guidelines for treating dyslipidemia?

AnswerA No therapy recommended at this time.B Initiate therapy with Crestor 10 mgC Initiate therapy with Zocor 10 mgD Initiate therapy with Zocor 40 mgE Initiate therapy with Lipitor 40 mgeLack of vitamin B1 is a common occurrence in patients with which of the following conditions?AnswerA HypothyroidismB Ankylosing spondylitisC Pernicious anemiaD AlcoholismE Graves diseased A patient with psychosis and a long history of depression and alcohol abuse had a lack of noticeable benefit from trials of risperidone and haloperidol. Clozapine was initiated. As the dose was being titrated upward, the patient had a seizure. The dose was decreased slightly. The patient had a second seizure that night. Which of the following statements is correct?AnswerA The event should be reported to the state Board of Pharmacy which monitors severe drug reactions.B The patient must have taken an overdose of the clozapine.C The physician should be disciplined for prescribing this drug to a patient with these medical conditions.D The pharmacist should not have dispensed the second refill; this is a medication error.E The patient experienced an adverse drug reaction to clozapine.eMultaq is a CYP 3A4 substrate. Which of the following statements concerning this drug is correct?AnswerA A patient using this antiarrhythmic should limit the use of grapefruit to fewer than 1 or 2 servings weekly.B The concentration of Multaq will decrease when given concomitantly with systemic azole antifungals.C The generic name of Multaq is dronedarone.D Multaq has a lower risk of liver damage than amiodarone and a higher risk of contributing to thyroid disorders.E Multaq has a low risk of liver damage.cAn investigational drug has the following pharmacokinetic parameters:

Bioavailability: 82%Clearance: 9.35 L/hrVolume of distribution: 87 L

What is the half-life of the investigational drug? Round to the nearest tenth. Enter the number only. Do not enter units or commas. Include a leading zero when the answer is less than one. 6.4Which of the following antipsychotics are available in oral formulations suitable for patients with difficulty swallowing or for use in patients who might try to hide the medicine in their mouth in order to spit it out? (Select ALL that apply.)AnswerA ClozapineB OlanzapineC RisperidoneD QuetiapineE FluphenazineabcA patient is taking 1 calcium carbonate tablet once daily. Each tablet weighs 1,500 milligrams (non-elemental total weight). She wishes to switch the calcium carbonate tablets for the calcium citrate form. She reports that her physician said that the "stomach absorption of the citrate form is better." The patient requires the same minimum amount of calcium supplementation daily. How many 1,000 milligrams citrate tablets (non-elemental total weight) will she need to provide a similar daily dose? Round to the nearest tablet. Enter the number only3A medication has an elimination rate constant of 0.016 hr^-1 and a clearance of 2.56 ml/hr. What is the half-life of the medication? Round to the nearest hour. Enter the number only. Do not enter units. 43Of the drugs below used to treat rheumatoid arthritis, which drugs cannot be used for patients with advanced heart failure?AnswerA EtanerceptB MethotrexateC HydroxychloroquineD SulfasalazineE LeflunomideaLiraglutide once daily was compared with exenatide twice daily for type 2 diabetes in a 26-week randomized open-label trial. What does it mean that the trial was open-label?AnswerA The patients could select their own dose based on their blood glucose values.B Neither the doctor nor the patients knew which patients received which treatment.C Both the doctor and the patients knew which drug the patients received.D The patients would start on one drug for 13 weeks, then switch to the other drug for 13 weeks.E The trial was observational in nature.cTom is picking up a prescription for pancreatic enzymes for his son. How should the pharmacist counsel him regarding administration of pancreatic enzymes? (Select ALL that apply.)AnswerA Give half of the dose before eating and the other half of the dose 1 hour after eating each meal.B Give this medication twice daily: first thing in the morning and at bedtime with a full glass of water.C Ideally the child should swallow the capsules whole, with meals and snacks, right before eating.D If your child has difficulty swallowing the capsules, you can sprinkle them into his mouth and have him swallow right away.E Give 1/2 his normal dose with snacks.ceA bottle is labeled 0.89 PPM of Drug X. How many liters of this solution will contain 5 mg of Drug X? Round to the nearest tenth. Enter the number only. Do not enter units or commas. 5.6QuestionWhich of the following statements is true about the progestin-only daily contraceptive pill?AnswerA One of the brand names is Microgestin.B The progestin-only pill should be avoided during lactation.C If more than 3 hours have elapsed from regular dosing schedule, a back-up method is needed for 48 hours after taking the late pill.D The first pack must be started on the Sunday after menstruation begins.E The progestin-only pill is more effective than estrogen/progesterone combinations.cA patient has been prescribed cyclobenzaprine. Which of the following statements is correct?AnswerA Cyclobenzaprine acts as a central alpha-1 agonist.B Abrupt withdrawal of agents in this class may result in severe rebound spasticity.C The brand name of cyclobenzaprine is Soma.D The brand name of cyclobenzaprine is Flexeril.E The usual dose of cyclobenzaprine is 20 mg PO TID.d Joan has diabetes with microalbuminuria. The blood pressure today is 162/91 mmHg. She has no other co-morbid conditions. Which drug would be an appropriate first-line antihypertensive for this patient?AnswerA ValsartanB MetoprololC HydrochlorothiazideD LabetololE DiltiazemaWhich of the following statements concerning Zyvox is true?AnswerA Zyvox is part of the streptogramin class of antibiotics.B Zyvox provides coverage against most gram-negative bacteria.C The Zyvox oral suspension should be refrigerated.D Zyvox should not be used within 2 weeks of MAO inhibitors.E The Zyvox dose should be adjusted with moderate renal impairment, defined as a CrCl < 60 mL/minute.d1/12/15Thalitone 12.5 mg dailyNorvasc 10 mg daily Prinivil 10 mg daily (dispense 30 with no refills)Celexa 20 mg daily - discontinueCymbalta 40 mg daily (dispense 30 with no refills)MacroBID 100 mg BID x 7 daysOTC aspirin 81 mg dailyOTC calcium and vitamin D encouraged

2/3/15Obtain discharge medication summary from hospital

Height, Weight and Vital Signs: Height: 5'10" Weight: 167 lbs Vitals: BP: 152/83 HR: 85 RR: 15 Temp: 98.9 F O2 Sat: 98% Pain: 0/10

Question:During TZ's hospitalization, she was started on an oral anticoagulant for stroke prevention. What drug interaction exists between anticoagulation therapy and the medications TZ was taking at her last clinic visit on 1/12/15.AnswerA Thalitone reduces the efficacy of anticoagulation therapy.B Celexa inceases bleeding risk when used concurrently with anticoagulants.C Prinivil inceases bleeding risk when used concurrently with anticoagulants.D Norvasc reduces the efficacy of anticoagulation therapy.E Thalitone inceases bleeding risk when used concurrently with anticoagulants.bWhich of the following drugs will lower the seizure threshold and increase the risk of seizures in susceptible patients? (Select ALL that apply.)AnswerA DemerolB MethyldopaC ClozarilD LariamE WellbutrinacdeA recipe for an oral product reads as follows: Dissolve five 25 mg tablets in 50 mL of solution." What is the ratio strength (% w/v) of the final product? Round to the nearest whole number. Type in only the second number (x) of the ratio strength (1:x). Do not enter units. 400Which of the following statements are true about HIV pre-exposure prophylaxis? (Select ALL that apply.)AnswerA An individual who does not have HIV can take one Truvada tablet daily to reduce his risk of becoming infected.B An individual who has HIV can take one Atripla tablet daily to reduce his risk of transmitting the virus.C The individual must be at a very high risk for acquiring HIV in order to be a candidate for pre-exposure prophylaxis.D The HIV test must be taken weekly when using pre-exposure prophylaxis therapy.E Once treatment is initiated, the individual no longer needs to follow safe sex practices.acA 43 year-old female patient with type 2 diabetes and normal renal function started taking Januvia 100 mg daily in the morning. Which of the following is correct regarding Januvia? (Select ALL that apply.)AnswerA The chemical name is saxagliptin.B It is an incretin enhancer.C It is an alpha-glucosidase inhibitor.D It is weight neutral.E It can cause a rash.bdeJoanna is 76 year old woman who will be starting Enablex for overactive bladder. Dry mouth is a common side effect of many medications used for this condition. Which statement below is correct regarding dry mouth and this class of medications?AnswerA Using long-acting formulations, topical gels or patches (versus immediate release oral formulations) can help reduce dry mouth.B Dry mouth is a cholinergic side effect of drugs used for overactive bladder.C Counsel patients to suck on lollipops or sugar candy to keep mouth moist.D Dry mouth lowers the incidence of dental caries, which is beneficial, but is uncomfortable.E Over-the-counter diphenhydramine can lessen this side effect but cannot be recommended in patients with aA young adult female patient has been prescribed Tylenol #3 in the emergency department. Choose the correct statement concerning Tylenol #3. AnswerA This medication contains acetaminophen and hydrocodone.B Tylenol #3 is a DEA Schedule II drug.C She will produce higher amounts of morphine if she is a 2D6 rapid metabolizer, which could be harmful if she is breastfeeding.D Tylenol #3 has a low degree of GI side effects compared to drugs such as Vicodin or Norco.E Codeine is the drug of choice in children post-tonsillectomy.cWhich of the following agents is associated with modest weight loss? AnswerA TopamaxB LyricaC TrileptalD KeppraE NeurontinaA pharmacist receives a prescription for Ciloxan. Which of the following conditions is Ciloxan used to treat? AnswerA Gray syndromeB CytomegalovirusC CellulitisD Bacterial conjunctivitisE Septic arthritisdQuestion: DD is started on Solu-Medrol 20 mg IV Q8H for the first 48 hours of her hospitalization. Her oxygen saturation has gradually improved and her physician has asked the pharmacist to convert her to the equivalent dose of oral prednisone. What is the equivalent daily dose of oral prednisone?

AnswerA 15 mgB 25 mgC 48 mgD 75 mgE 100 mgdJames is a 56 year old male who has brings a prescription for tapentadol ER to the pharmacy. Choose the correct statement about this medication:AnswerA The brand name is Nucynta ER.B The brand name is Oxecta ER.C Males may consume a maximum of 3 alcoholic drinks and females a maximum of 2 alcoholic drink per day while taking tapentadol ER or other similar drugs.D Tapentadol is a DEA schedule III drug.E Tapentadol should be avoid in patients with aspirin sensitivity or allergy.a Doris is a 67 year-old female who has been prescribed weekly alendronate for osteoporosis prevention. Which of the following dosing recommendations is correct for this patient?AnswerA 5 mg PO weeklyB 10 mg PO weeklyC 35 mg PO weeklyD 70 mg PO weeklyE 140 mg PO weeklycThe pharmacy receives the following parenteral nutrition order. What percentage of the total calories is represented by the protein component? Round to the nearest whole number. Enter the number only. Do not enter the percentage sign.

Item/Quantities:

Dextrose 280 gramsAmino acids 75 gramsSodium chloride 38 mEqSodium acetate 16 mEqPotassium 40 mEqMagnesium sulfate 10 mEqPhosphate 21 mmolCalcium 4.45 mEqMultivitamins (MVI) 10 mLTrace elements 1 mLFamotidine 10 mgRegular insulin 40 units

Sterile water qs ad 980 mL24Mandy is a 22 year old graduate student. She has suffered from bouts of depression for as long as she can remember. Mandy has failed trials of citalopram and sertraline. The prescriber has written her a prescription for bupropion. Which of the following are side effects or potential adverse effects of bupropion? AnswerA Seizures, impotence, and hyponatremiaB Dry mouth, seizures, hepatic failure (especially during the first 6 months of therapy)C Hyponatremia, insomnia, impotenceD Dry mouth, insomnia, and seizuresE Impotence, insomnia, hepatic failure (especially during the first 6 months of therapy)dMark is a 28 year old male who was in a motor vehicle accident, resulting in moderate to severe pain. The doctor advised him to alternate doses between the prescription Vicodin 5 mg with over-the-counter extra-strength acetaminophen 500 mg. He has been using 5-6 Vicodin tablets and 5-6 acetaminophen tablets per day. Select the correct statement concerning risk to the patient from this medication regimen.AnswerA The recommendation to alternate Vicodin with acetaminophen is unsafe and should not be continued at any dosing regimen.B The primary risk to the patient is drowsiness.C The primary risk to the patient is addiction potential.D The primary risk to the patient is kidney damage.E The primary risk to the patient is liver damage.eJack is beginning amiodarone therapy for ventricular fibrillation. His other medical conditions include heart failure, two myocardial infarctions, long-standing hypertension, glaucoma and benign prostatic hypertrophy. Jack's daily medications include digoxin 0.25 mg daily. What suggestion should the pharmacist make to Jack's healthcare provider?AnswerA The dose of digoxin should be decreased by up to 50% due to the inhibition effect of amiodarone.B Amiodarone will increase the metabolism of digoxin.C The dose of digoxin should be increased by 30-50% in most patients beginning concurrent amiodarone therapy.D The amiodarone should be switched to dronedarone, which is considered the preferable agent when using digoxin concurrently.E Amiodarone is contraindicated in patients receiving digoxin.aWhich of the following injection sites and type is correct for administration of Lovenox?AnswerA Thigh, subcutaneousB Triceps (fatty tissue), subcutaneousC Abdomen, subcutaneousD Abdomen, intramuscularE Deltoid, intramuscularcWhich of the following is considered a high-intensity statin? AnswerA Crestor 20 mgB Pravachol 40 mgC Mevacor 40 mgD Lescol 40 mgE Livalo 1 mga Which of the following statements concerning the influenza vaccine is correct?AnswerA ACIP recommends that everyone receive an annual influenza vaccine from the ages of 2 years and older.B Healthcare personnel, even if healthy, should receive the flu shot and not the live flu intranasal formulation.C The completely egg-free flu vaccine product is called Flucelvex.D The completely egg-free flu vaccine product is called Flublok.E Healthy patients ages 12 months to 55 years can receive the live flu vaccine.dA female patient is in the critical care step-down unit. She has been diagnosed with a pulmonary embolism and will receive anticoagulation therapy. The patient was admitted three days previously for mitral valve replacement surgery. She will receive 7,500 units of heparin in 250 mL 0.9% NaCl to be infused at a rate of 1,800 units/hour. The infusion set delivers 20 drops per milliliter. How many drops does the patient receive per minute? Enter the number only. Do not enter units or commas.20Mary was feeling poorly and went to the physician, who ordered a thyroid panel. The lab results come back with a high TSH and low T4. What is the correct interpretation of these results? Choose the correct statement.AnswerA Mary has hyperthyroidism, as evidenced by the high TSH.B Mary has hypothyroidism, as evidenced by the low T4.C Mary should be treated with potassium iodide.D Mary's thyroid status cannot be determined without the T3 value.E Mary has Grave's disease.b What are possible complications with long-term phenytoin therapy? (Select ALL that apply.)AnswerA HirsutismB Skin thickening (children)C Shrinkage/atrophy of dental gum tissueD OsteoporosisE HypertensionabdNancy is a business executive being seen in the Family Medicine Clinic for "really bad sadness and feeling really depressed" that she reports occur every month, during the week prior to her menstrual cycle. She has an important presentation coming up next month with the senior vice president of her company, which is scheduled the week before her period. She is asking for some medicine that will help her avoid becoming depressed every month. Which statement is true regarding the treatment of Pre-Menstrual Dysphoric Disorder (PMDD)?AnswerA PMDD can be treated, in most cases, with Abilify.B Sarafem is indicated for PMDD and contains the SSRI sertraline.C PMDD can be treated effectively with the mini-pill in women who are not candidates for estrogen therapy.D PMDD can be treated, in most cases, with light therapy.E She can be treated with Sarafem 20 mg PO Q AM, starting 14 days before the expected date of her next period (repeat with each cycle).eJohn is a 50 year old male who comes into the pharmacy with a prescription for Advair Diskus 250/50, one inhalation BID. Choose the correct statement/s concerning this drug. (Select ALL that apply.)AnswerA Advair 250/50 contains 250 mcg fluticasone and 50 mcg salmeterol.B Advair 250/50 contains 250 mcg salmeterol and 250 mcg fluticasone.C An equivalent HFA dose is Advair HFA 230/21, taken two puffs BID.D An equivalent HFA dose is Advair HFA 250/50, taken one puff BID.E If also using a rescue inhaler at the same time, the patient should be instructed to use the rescue inhaler first.aeOmeprazole can increase the risk of which of the following events? (Select ALL that apply.)AnswerA Pneumonia in hospitalized patientsB Urinary tract infectionsC Heart attacksD FracturesE C. difficile infectionsadeWhat is an appropriate starting dose for a patient beginning metformin for type 2 diabetes?AnswerA 250 mg twice dailyB 500 mg twice dailyC 1000 mg twice dailyD 2.5 mg once dailyE 5 mg once dailybAn 84 year old female patient is admitted to the hospital. She weighs 145 pounds and is 5 foot 6 inches tall. Her admission labs are reported below. What is the patient's creatinine clearance using her ideal body weight? Round to the nearest whole number. Enter the number only. Do not enter units.

Labs:Na (mEq/L) = 142 (135 145) K (mEq/L) = 4.4 (3.5 5) Cl (mEq/L) = 102 (95 103) HCO3 (mEq/L) = 26 (24 30) BUN (mg/dL) = 18 (7 20) SCr (mg/dL) = 1.1 (0.6 1.3) Glucose (mg/dL) = 98 (100 125) 36Which vaccine contains Hepatitis A and Hepatitis B in a combination shot?AnswerA CombivaxB ComvaxC TwinrixD VarivaxE AfluriacQuestionWhich of the following statements concerning insulins are correct? (Select ALL that apply.)AnswerA Glulisine is a long-acting insulin.B Novolog 70/30 contains 70% longer-acting insulin and 30% shorter-acting insulin.C Lispro and Apidra are rapid-acting insulins.D The common insulin concentration is 200 units in each mL.E Aspart is a rapid-acting insulin called Humalog.bcJordan has anemia caused by end stage renal disease (ESRD). Which of the following statements is true concerning treatment of this type of anemia?AnswerA Patients receiving Procrit should be warned of the risk of hypotension.B Treatment with erythropoietin stimulating agents (ESAs) should continue until the hemoglobin level is > 12 g/dL.C ESAs should only be used to treat anemia in ESRD when the hemoglobin level is < 10 g/dL.D Aranesp carries a boxed warning for angina.E Aranesp can improve certain types of blood cancer, but is not useful for other types of cancer.cA patient receiving ramipril and a potassium-sparing diuretic is hospitalized with heart rhythm problems consistent with hyperkalemia. Which of the following drugs should be administered to counteract the effects of potassium on the heart?AnswerA Digoxin, InjectionB DigiFab, InjectionC Kayexalate, RectalD Calcium, InjectionE Glucagon, InjectiondThe pharmacist has an order for heparin 20,000 units in 500 mL D5W to infuse at 2,000 units/hour. What is the infusion rate in mL/hour? Enter the number only. Do not enter units.50A patient is receiving 500 mL of sodium chloride irrigation solution labeled 327 mOsm/L. The pharmacist needs to reduce the osmolarity of the irrigation solution to 225 mOsm/L by diluting with sterile water. How much sterile water should be added to the bag? Round to the nearest milliliter. Enter the number only. Do not enter units or commas.

227What is the mechanism of action of ranitidine?AnswerA Neutralizes pepsin via hydroxylation.B Neutralizes acid in a buffering reaction (producing salt and water).C Reversibly inhibits the histamine-2 receptors on the parietal cells.D Irreversibly binds to parietal cells which blocks the final step in acid production.E Irreversibly inhibits the histamine-2 receptors on the parietal cells.c George is a 225 pound male who has been taking ramipril 5 mg daily and chlorthalidone 25 mg daily for hypertension and morphine 60 mg daily for chronic pain. The physician wishes to change the morphine to a fentanyl patch. Which statement is correct?AnswerA The patient is not a candidate for fentanyl patches.B Fentanyl patches can only be used in patients taking 100 mg of morphine equivalent daily; the patient does not qualify for this reason.C The fentanyl patch dosing schedule should begin at one patch every 3 days.D When the fentanyl patch is removed it should be folded in half (sticky sides together) and discarded in the trash, assuming that there are no children in the household.E The morphine should be discontinued the same morning as the patch is started to avoid risk of overdose.cWhen TZ went to the hospital for chest pain on 1/15/15, she was very anemic (Hgb 10.4 g/dL). A Coombs Test was ordered. What medication prescribed in the Family Medicine Clinic on 1/12/15 was part of the drug-induced differential diagnosis her providers were considering based on the test they ordered? AnswerA PrinivilB ThalitoneC MacrobidD CymbaltaE NorvasccMae is an 82 year old woman in a skilled nursing facility taking chlorpromazine for dementia-related psychosis. The nurse notices she is making a smacking motion with her lips and unusual movements with her eyes. Which of the following statements is correct?AnswerA Because of her age, she is at an increased risk of the drug causing death due to renal failure.B She is experiencing a form of akathesia, which can be corrected by a reduction in dose.C She is experiencing a form of tardive dyskinesia. This side effect is more likely because of her age and gender.D She is experiencing a form of tardive dyskinesia. This side effect is more likely in young males.E She is experiencing a form of dystonia. This side effect is more likely in young males.cA patient comes to the pharmacy with a prescription for Harvoni. What are the components of Harvoni and what is it used to treat?AnswerA Ledipasvir and simeprevir; Hepatitis BB Ledipasvir and sofosbuvir; Hepatitis CC Simeprevir and sofosbuvir; Hepatitis CD Ribavirin and simeprevir; Hepatitis BE Lamivudine and ledipasvir; Hepatitis CbWhich antipsychotic is most likely to present a problem in a patient already taking a drug that prolongs the QT interval?AnswerA RisperdalB GeodonC InvegaD LatudaE Saphrisb parenteral nutrition order is received for 725 mL of D50%, 890 mL of Aminosyn 8.5%, 500 mL of 10% lipid emulsion and 30 mL of electrolytes. How many calories will this patient receive from dextrose? Round to the nearest whole number. Enter the number only. Do not enter units or commas. 1233A medical resident calls the pharmacy to ask how a patient should be converted from warfarin to Pradaxa. What advice should the pharmacist offer?AnswerA Discontinue warfarin. Start Pradaxa 2 hours before the next scheduled dose of warfarin.B Discontinue warfarin. Start Pradaxa when the INR is less than 1.5.C Discontinue warfarin. Start Pradaxa when the INR is less than 1.D Decrease the dose of warfarin by 50%, then discontinue warfarin and start Pradaxa when the INR is less than 1.5.E Discontinue warfarin. Start Pradaxa when the INR is less than 2.eWhat is the safest and most practical use of prescription stimulant agents for weight loss?AnswerA Use only short-term, in addition to changes in diet and increased physical activity and only for patients with a BMI of 27-30 who have at least one weight-related comorbidity.B Useful in patients who will not engage in a regular diet and physical activity program.C Useful in patients with cardiovascular disease who require fast weight loss.D Useful long-term to help maintain weight loss in patients who cannot decrease caloric intake by other means.E Useful in patients who experience lethargy or are frequently tired.aA pharmacist is preparing an injection of the Zostavax vaccine. Which of the following statements concerning Zostavax are correct? (Select ALL that apply.)AnswerA Use a needle at least 1.5" long in overweight patients.B The vaccine must be reconstituted prior to use with normal saline.C Do not administer if the patient has an allergy to neomycin or gelatin.D Zostavax is a live vaccine and must not be used in patients with severe immunosuppression.E This vaccine is administered subcutaneously in the fatty tissue over the triceps.cdeA pharmacist adds 600 mg of a drug to 250 mL of 0.9% sodium chloride. If the solution is administered at 0.8 mL per minute, how many milligrams of the drug will the patient receive in 1 hour? Do not round the answer. Enter the number only. Do not enter units or commas.115.2A patient brings in a new prescription for clarithromycin to your pharmacy. He says he is allergic to erythromycin. Which questions would you ask him to determine if the clarithromycin prescription can be filled as written? (Select ALL that apply.)AnswerA Did you receive any treatment as a result of your reaction to erythromycin?B Is anyone else in your family allergic to erythromycin?C Have you ever taken Biaxin in the past?D What happened when you took erythromycin? Did you have nausea, or stomach upset, or swelling in any part of your mouth or face, or any trouble breathing easily?E Have you ever taken Azactam in the past?acdA patient is receiving ribavirin as part of her hepatitis C regimen. Which of the following are boxed warnings for ribavirin? AnswerA Significant teratogenic effects and lactic acidosisB Lactic acidosis and hemolytic anemiaC Hemolytic anemia and vesicant potentialD Significant teratogenic effects and hemolytic anemiaE Vesicant potential and significant teratogenic effectsdWhich of the following medications represents a common over-the-counter agent used to treat opioid-induced constipation?AnswerA Psyllium or methylcelluloseB SennosidesC LubiprostoneD OsmoPrepE PicosulfatebA 4 mL vial is labeled potassium chloride (4 mEq/mL). How many grams of potassium chloride are present in the vial? (MW of KCl = 74.5). Round to the nearest tenth. Enter the number only. Do not enter units or commas.1.2Chief Complaint: I'm in serious pain

History of Present Illness: MG is a 62 year old African American male admitted to the hospital on 2/10/15 with a 3 day history of worsening left foot pain. He rates the pain a "15 out of 10". He thinks he has a scab on the bottom of his foot, but cannot see the bottom of his foot. He knows there has been bloody discharge on his sock. MGs past medical history is significant for hypertension, type 2 diabetes, hypothyroidism, and chronic back pain. He does not drink alcohol and does not smoke.

Allergies: sulfa and Cipro

Medications: Hyzaar 50/12.5 mg daily, Norvasc 10 mg daily, Byetta 10 mcg SC BID, metformin 500 mg BID, Synthroid 100 mcg daily, Norco 5/325 mg every 6 hours PRN.

Physical Exam / Vitals: Height: 511 Weight: 327 pounds BP: 146/89 mmHg HR: 92 BPM RR: 16 BPM Temp: 100.9F Pain: 10/10

Gen: Obese male in moderate distressHeart: Regular rate and rhythmLungs: Clear to auscultationGI: Normal bowel soundsExtremities: Large area (7 cm x 9 cm) oozing pus and blood on sole of left foot. Some surrounding cellulitis (mild). Decreased sensation and decreased pulses bilaterally.

Labs on 2/10/15:Na (mEq/L) = 137 (135 145) WBC (cells/mm3) = 14.2 (4 11 x 10^3) K (mEq/L) = 3.5 (3.5 5) Hgb (g/dL) = 13.6 (13.5 18 male, 12 16 female)Cl (mEq/L) = 99 (95 103) Hct (%) = 41.1 (38 50 male, 36 46 female)HCO3 (mEq/L) = 28 (24 30) Plt (cells/mm3) = 301 (150 450 x 10^3)BUN (mg/dL) = 18 (7 20) AST (IU/L) = 29 (10 40) SCr (mg/dL) = 1.2 (0.6 1.3) ALT (IU/L) = 25 (10 40) Glucose (mg/dL) = 254 (100 125) Albumin (g/dL) = 3.1 (3.5 5) Ca (mg/dL) = 7.2 (8.5 10.5) A1C (%) = 10.2 Mg (mEq/L) = 1.1 (1.3 2.1) PO4 (mg/dL) = 2.3 (2.3 4.7) Tests on 2/10/15: Radiology of left foot: soft tissue swelling, but no evidence of bone involvement. Negative for osteomyelitis.

Plan:Admit to floor for incision and drainage of left foot wound, management of infection, and management of chronic conditions.

Question: While awaiting the incision and drainage, the infectious disease specialist would like to treat MG with antibiotics covering gram-positive skin pathogens, gram-negatives (including Pseudomonas), and anaerobes. Which of the following IV regimens should the pharmacist recommend for MG?

AnswerA Unasyn monotherapyB Clindamycin + CiproC Rocephin + FlagylD Zosyn monotherapyE INVanz monotherapydWhich of the following are appropriate patient counseling points for the Combivent Respimat? (Select ALL that apply.)AnswerA Common side effects include cough and dry mouth.B To take an inhalation, turn the base, open the cap, and press the dose-release button.C Shake the inhaler prior to taking an inhalation.D The Combivent Respimat needs to be primed prior to use.E Take 1 inhalation four times daily.abdeA pharmacist has calculated that a patient requires 27 mmol of phosphate and 75 mEq of potassium for today's PN. How many milliliters of potassium phosphate and how many milliliters of potassium chloride will be required? [K-Phos contains 3 mmol Phosphate with 4.4 mEq K+ per mL, and KCl contains 2 mEq K+ per mL]AnswerA 9 mL KPO4 and 17.7 mL KClB 17.7 mL KPO4 and 9 mL KClC 28.5 mL KPO4 and 9 mL KClD 6.1 mL KPO4 and 37.5 mL KClE 37.5 mL KPO4 and 6.1 mL KClaA patient picks up a new prescription for tacrolimus capsules from the pharmacy. Which of the following points should not be included in the counseling session? AnswerA This is the preferred transplant drug because it does not require regular lab (blood) monitoring tests.B You may experience headaches, nausea, diarrhea and/or tremor.C Your weight, temperature and blood pressure (and blood sugar if you have diabetes) should be monitored.D You are at risk for infection; report at once if you develop fever or feel weak and tired. This drug increases the risk of developing lymphoma.E Tacrolimus is also called Prograf. There is a different formulation of this drug called Astagraf XL.aEtanercept carries a boxed warning for which of the following conditions? (Select ALL that apply.)AnswerA TeratogenicityB Increased risk of serious infectionsC High risk of severe SJS/TEN rashD Increased risk of lymphomasE Reactivation of latent tuberculosisbdeTed has Parkinson Disease and uses Requip and Parcopa to control his symptoms. Ted has been exhibiting some disturbing symptoms of psychosis. Which is the most appropriate antipsychotic for Ted?AnswerA HaldolB RisperdalC FanaptD SeroquelE FluphenazinedA 58 year-old male patient complains of impotence. He has severe liver impairment but states he is healthy enough for sexual activity. The physician prescribed him sildenafil 50 mg, 1 hour prior to sexual activity. Is this starting dose appropriate?AnswerA Yes, the dose is correct.B No, he should begin sildenafil at 25 mg, 1 hour prior to sexual activity.C No, he should begin sildenafil at 100 mg, 1 hour prior to sexual activity.D It would be preferable for him to use the alprostadil injection.E It would be preferable for him to use vardenafil.bWhich of the following drugs is not available in a suppository formulation?

AnswerA ClotrimazoleB AlprostadilC AcetaminophenD MesalamineE PhenazopyridineeA patient with sickle cell disease has been using Desferal, but is tired of the infusions. What can the pharmacist tell the patient about Exjade? Choose the correct statement.AnswerA Exjade has less interactions in the gut than Desferal.B Exjade can be used with severe renal impairment while Desferal cannot.C Exjade is taken 30 mins before eating on an empty stomach. It can cause gastrointestinal hemorrhage.D Exjade is taken with meals to reduce risk of gastrointestinal hemorrhage.E Exjade is an oral medication, but Desferal is better tolerated even though it is given by injection.cQuestion: DD is discharged on a new insulin regimen consisting of Levemir and Apidra pens. How long are these pens stable at room temperature when in use?

AnswerA Levemir 14 days, Apidra 28 daysB Levemir 28 days, Apidra 28 daysC Levemir 42 days, Apidra 42 daysD Levemir 42 days, Apidra 28 daysE Levemir 30 days, Apidra 10 dayseSusan has unsuccessfully tried various nicotine replacement products to quit smoking. She is interested in trying Chantix. Which of the following statements is correct about Chantix?AnswerA The generic name is varenicline.B The generic name is bupropion.C It is a dopamine antagonist.D It can also be used for patients with depression.E Chantix does not cause much somnolence, so is a good choice for people who need to be alert at work (air traffic controllers, commercial truck drivers).aQuestionRiva has cystic fibrosis and is currently taking pancreatic enzyme products. Which of the following are FDA approved pancreatic enzyme formulations? (Select ALL that apply.)AnswerA BetapaceB ZohydroC ZenpepD ViokaceE CreoncdeA patient weighing 115 kg is to receive 0.4 mg/kg per day amphotericin B by IV infusion. The product is reconstituted and diluted to 0.1 mg/mL. What volume (mL) of solution is required to deliver the daily dose? Enter the number only. Do not enter units or commas. 460In a pivotal clinical trial in 1,000 patients for the new antibiotic Super X, there were four cases of Stevens Johnson syndrome in the treatment group and 1 in the placebo group. There were 500 people in each treatment arm. What is the relative risk of Stevens Johnson syndrome? Enter the number only. Do not enter units.4Counseling points for Aranesp should include which of the following?AnswerA Seek immediate medical attention if you have pale stools.B Shake the vial well before drawing up your dose.C You can pre-fill several syringes provided you store them in the refrigerator.D Inject into the deltoid or buttock muscle.E Make sure you monitor your blood pressure regularly; this medication can increase blood pressure.eWhich of the following bacterial organisms is not covered by ertapenem? AnswerA StaphylococciB StreptococciC AcinetobacterD E. coliE H. influenzaecA 52-year-old patient is receiving a PN containing 26 mmol of phosphate and 12 mEq of calcium in a total volume of 2,100 mL. There are 2 mEq PO4/mmol. The combined calcium and phosphorus in a PN should not exceed 45 mEq/L. What is the combined calcium and phosphorus in this PN order and is it considered safe?AnswerA 64 mEq/L, noB 30.5 mEq/L, yesC 32 mEq/L, yesD 72 mEq/L, noE 27.4 mEq/L, yesbA pharmacist needs to prepare 180 grams of a 15% lidocaine ointment. The pharmacist has 2% and 20% lidocaine ointments in stock. How many grams of each ointment will the pharmacist need to prepare this prescription?AnswerA 50 g of 2% and 130 g of 20%B 50 g of 20% and 130 g of 2%C 20 g of 2% and 200 g of 20%D 110 g of 2% and 80 g of 20%E 80 g of 2% and 110 g of 2%aWhat is the mechanism of action of Arixtra?AnswerA Direct thrombin inhibitorB Vitamin K antagonistC Blocks fibrinogen binding to GPIIb/IIIa receptors on plateletsD Synthetic pentasaccharide that binds to antithrombin to selectively inhibit Factor XaE Inhibits P2Y12 receptor on plateletsdWhich statements concerning the prostaglandin analogs used for glaucoma are correct? (Select ALL that apply.)AnswerA The patient should keep all in-use bottles of any of the prostaglandin analogs in the refrigerator.B The patient can keep the bottle of the prostaglandin analog in use at room temperature.C The unused bottles of prostaglandin analogs should be kept frozen.D In the pharmacy, the intact bottles of Lumigan are stored in the refrigerator.E In the pharmacy, the intact bottles of Xalatan are stored in the refrigerator.be Jack developed trouble breathing and hypotension when receiving morphine by injection in the hospital. Which drug would not pose a risk for cross-reaction in a patient with an allergy to morphine?AnswerA OxycodoneB FentanylC HydromorphoneD HydrocodoneE OxymorphonebAt 2000 hours, the concentration of Drug A was measured at 52.6 mcg/mL. The next morning, at 0800 hours, the concentration was measured at 26.3 mcg/mL. What is the half-life, in hours, of drug A? Enter the number only. Do not enter units or commas. Include a leading zero when the answer is less than one.12A pharmacist will counsel a patient who is beginning lamotrigine therapy for bipolar disorder. Which of the following statements should the pharmacist share with the patient? (Select ALL that apply.)AnswerA This medication can cause serious skin reactions, including a very severe rash, especially if the dose is increased too quickly. Make sure you increase the dose gradually as instructed.B One of the brand names for this medicine is Keppra.C One of the brand names for this medicine is Stavzor.D Do not stop taking this medicine abruptly. The dose will need to be slowly decreased by your physician.E This medication has few drug interactions and is safely used with most other seizure drugs. A patient with atrial fibrillation was confused about his digoxin dose and took too many tablets, several days in a row. On admission to the medical floor, his digoxin level was 4.8 ng/mL at 1300. He was not experiencing signs or symptoms of toxicity, so his providers will wait until the patient's digoxin level decreases to 1.5 ng/mL before restarting the digoxin maintenance dose. The patient's digoxin half-life is estimated by the pharmacist to be 42 hours. How many hours will it take for the patient's digoxin level to fall to 1.5 ng/mL so that his maintenance dose can be restarted? Round to the nearest hour. Enter the number only. Do not enter units.70Which of the following dosing regimens would be appropriate for a patient using niacin extended-release?AnswerA Start with of a 500 mg tablet with dinner, can increase weekly up to 6 grams daily (divide the doses and take with meals).B Start at 500 mg QHS x 4 weeks (after a light snack), followed by 1,000 mg QHS x 4 weeks, can increase to 2,000 mg QHS.C Start with 1,000 mg tablet with dinner, can increase weekly up to 6 grams daily (divide the doses and take with meals).D Start with 2,000 mg tablet with dinner, can increase weekly up to 5 grams daily (divide the doses and take with meals).E Start at 1,500 mg QHS x 4 weeks (after a light snack), followed by 2,000 mg QHS x 4 weeks, can increase to 5,000 mg QHS.bChief Complaint: I'm out of refills on my inhaler and having trouble with my breathing

History of Present Illness: DD is a 24 year old Caucasian female who presents to the emergency room on 1/29/15 because she ran out of refills on her inhaler about a week ago. Her past medical history is significant for type 1 diabetes and asthma. She lives with her son. She does not drink alcohol and does not smoke tobacco. She occasionally smokes marijuana.

Allergies: penicillin

Medications: Advair Diskus 250/50 mcg 1 inhalation BID, Mircette 1 tab daily, NovoLIN 70/30 14 units BID, and an OTC multivitamin daily.

Physical Exam / Vitals: Height: 52 Weight: 118 pounds BP: 118/74 mmHg HR: 95 BPM RR: 19 BPM Temp: 98.6F Pain: 0/10 O2 Sat: 92% on room air

Gen: Thin female, very anxiousHEENT: no nasal discharge, mild erythema of throatHeart: Regular rate and rhythmLungs: Wheezing heard bilaterallyExt: No skin lesions, good peripheral pulses bilaterally

Labs on 1/29/15:Na (mEq/L) = 142 (135 145) WBC (cells/mm3) = 11.7 (4 11 x 10^3) K (mEq/L) = 4.2 (3.5 5) Hgb (g/dL) = 14.3 (13.5 18 male, 12 16 female)Cl (mEq/L) = 102 (95 103) Hct (%) = 43.7 (38 50 male, 36 46 female)HCO3 (mEq/L) = 25 (24 30) Plt (cells/mm3) = 402 (150 450 x 10^3)BUN (mg/dL) = 7 (7 20) AST (IU/L) = 18 (10 40) SCr (mg/dL) = 0.7 (0.6 1.3) ALT (IU/L) = 22 (10 40) Glucose (mg/dL) = 176 (100 125) Albumin (g/dL) = 4.7 (3.5 5) Ca (mg/dL) = 9.2 (8.5 10.5) A1C (%) = 8.7 Mg (mEq/L) = 2.2 (1.3 2.1) PO4 (mg/dL) = 8.6 (2.3 4.7) Tests on 1/29/15: Chest X-ray: no infiltrate

Plan:Admit to floor for asthma exacerbation.

Question: What is the best action to take regarding DD's serum phosphorus?

AnswerA Initiate PhosLo 667 mg TID with mealsB Initiate Fosrenol 500 mg TID with mealsC Initiate Renagel 800 mg TID with mealsD Obtain an ultrasound of the kidneys. DD likely has undiagnosed chronic kidney disease.E Repeat the laboratory test. This finding is not consistent with DD's medical history or medications.eA cancer patient has developed anemia. Which of the following statements concerning the use of erythropoiesis stimulating agents (ESA) is correct?AnswerA The only acceptable time to use these agents in a patient with cancer is if the treatment goal is curative.B The only acceptable time to use these agents in a patient with cancer is if the hemoglobin level is < 12 g/dL.C The only acceptable time to use these agents in a patient with cancer is if the hemoglobin level is < 14 g/dL.D The blood pressure can fall very low and must be monitored in a patient using these agents.E These agents contribute to tumor growth and are no longer used commonly to treat anemia in patients with cancer.eWhich of the following are long-acting anticholinergics? (Select ALL that apply.)AnswerA Tudorza PressairB Atrovent HFAC Spiriva HandiHalerD Arcapta NeohalerE Breo ElliptaacBob was recently diagnosed with angina. His physician plans to prescribe nitroglycerin. Which of the following formulations will not be an option for the nitroglycerin prescription?AnswerA Sublingual sprayB Sublingual tabletsC Topical ointmentD Transdermal patchesE Rectal suppositoryeQuestion:During TZ's hospitalization, she was started on an oral anticoagulant for stroke prevention that requires no monitoring for efficacy. Which of the following medications meet this criteria? AnswerA Eliquis, Xarelto, PradaxaB Eliquis, Arixtra, warfarinC Lovenox, Xarelto, PradaxaD Fragmin, Pradaxa, argatrobanE Pradaxa, Xarelto, warfarinaQuestionAn order is received for a parenteral nutrition solution containing 1,340 mL of D20W, 630 mL of 15% amino acid, 210 mL of 20% lipid emulsion, and 30 mL of electrolytes. The 15% amino acid is on backorder, but the pharmacy has 8.5% amino acid solution on hand. How many milliliters of 8.5% amino acid solution will be required to compound the PN solution? Round to the nearest milliliter. Enter the number only. Do not enter units or commas.1112Which of the following statements concerning sumatriptan is correct?AnswerA Sumatriptan is contraindicated in patients with uncontrolled hypertension.B Sumatriptan is available as a needleless injection, intramuscular injection, and nasal spray.C If the migraine goes away and then returns, a second dose can be taken if 3 hours have elapsed since the first dose was administered.D The most common side effects of sumatriptan are throat/neck pressure and insomnia/trouble sleeping.E Imitrex nasal spray is suitable for patients with cerebrovascular disease due to lower risk of systemic toxicity.aThe pharmacy receives a prescription for 100 grams of 5% salicylic acid ointment. The pharmacy has a 30% salicylic acid ointment and petrolatum available. Using only these products, how much petrolatum should be used to compound the prescription? Express the answer in grams, rounded to the nearest tenth. Enter the number only. Do not enter units or commas. 83.3Parenteral nutrition is ordered for a patient in the cardiac care unit. The prescription is written for 750 mL of D50% w/v. The pharmacy is out of stock of D50% w/v and will substitute with D70% w/v. How many mL of D70% are required? Round to the nearest whole number. Enter the number only. Do not enter units or commas.536Charles is a 74 year-old male who has COPD with chronic bronchitis and emphysema. The physician has switched Charles from Atrovent to Spiriva in order to reduce the number of daily doses required. Which of the following is a correct statement regarding Spiriva?AnswerA Spiriva is formulated as a capsule that is placed into the HandiHaler device.B Spiriva contains ipratropium, which is an anticholinergic medication.C Spiriva is a long-acting beta-2 agonist.D Spiriva comes in a metered dose inhaler.E Spiriva is dosed 1 puff twice per day after breakfast and after dinner.aWhich of the following drugs are glycoprotein IIb/IIIa receptor antagonists? AnswerA Abciximab, ticagrelor, clopidogrelB Prasugrel, eptifibatide, abciximabC Eptifibatide, tirofiban, abciximabD Tirofibran, eptifibatide, ticagrelorE Clopidogrel, alteplase, tirofibancWhich of the following agents contains a long-acting beta agonist? (Select ALL that apply.) AnswerA SpirivaB Breo ElliptaC ArcaptaD SymbicortE TudorzabcdTom is picking up a prescription for pancreatic enzymes for his son. How should the pharmacist counsel him regarding administration of pancreatic enzymes? (Select ALL that apply.) A Give half of the dose before eating and the other half of the dose 1 hour after eating each meal.B Give this medication twice daily: first thing in the morning and at bedtime with a full glass of water.C Ideally the child should swallow the capsules whole, with meals and snacks, right before eating.D If your child has difficulty swallowing the capsules, you can sprinkle them into his mouth and have him swallow right away.E Give 1/2 his normal dose with snacks.ceJack is beginning amiodarone therapy for ventricular fibrillation. His other medical conditions include heart failure, two myocardial infarctions, long-standing hypertension, glaucoma and benign prostatic hypertrophy. Jack's daily medications include digoxin 0.25 mg daily. The pharmacist should call the prescriber with the following suggestion:AnswerA The dose of digoxin should be decreased by up to 50% due to the inhibition effect of amiodarone.B Amiodarone will increase the metabolism of digoxin.C The dose of digoxin should be increased by 30-50% in most patients beginning concurrent amiodarone therapy.D The amiodarone should be switched to dronedarone, which is considered the preferable agent when using digoxin concurrently.E Amiodarone is contraindicated in patients receiving digoxin.a193Choose the best definition of a cohort study:Choose the best definition of a cohort study: AnswerA A study that follows a group of patients with some commonality over a given time period.B A study used to estimate the relationship between an outcome of interest and population variables as they exist at a particular time.C A study in which the participants receive each treatment in a sequential order.D A pivotal clinical trial comparing a new drug to a placebo treatment group.E A pivotal clinical trial comparing a new drug to a the standard treatment used for that condition.aMary was feeling poorly and went to the physician, who ordered a thyroid panel. The lab results come back with a high TSH and low T4. What is the correct interpretation of these results? Choose the correct statement: AnswerA Mary has hyperthyroidism, as evidenced by the high TSH.B Mary has hypothyroidism, as evidenced by the low T4.C Mary should be treated with potassium iodide.D Marys thyroid status cannot be determined without the T3 value.E Mary has Graves disease.b